הבדלים בין גרסאות בדף "קוד:המספר של אוילר e"

מתוך Math-Wiki
קפיצה אל: ניווט, חיפוש
(יצירת דף עם התוכן "<latex2pdf> <tex>קוד:ראש</tex> \subsection{הגדרה} נגדיר את $x_n=(1+\frac{1}{n})^n$ . \underline{משפט:} קיים הגבול $\lim_{n\to \...")
 
מ (4 גרסאות יובאו)
 
(3 גרסאות ביניים של משתמש אחר אחד אינן מוצגות)
שורה 1: שורה 1:
<latex2pdf>
 
<tex>קוד:ראש</tex>
 
 
 
\subsection{הגדרה}
 
\subsection{הגדרה}
 
נגדיר את $x_n=(1+\frac{1}{n})^n$ .  
 
נגדיר את $x_n=(1+\frac{1}{n})^n$ .  
  
\underline{משפט:} קיים הגבול $\lim_{n\to \infty} x_n $ , לגבול הזה קוראים $e$.
+
\begin{thm}
 +
קיים הגבול $\lim_{n\to \infty} x_n $ , לגבול הזה קוראים $e$.
 +
\end{thm}
  
\underline{הוכחה:} נוכיח שהסדרה מונוטונית עולה וחסומה מלעיל.
+
\begin{proof}
 +
נוכיח שהסדרה מונוטונית עולה וחסומה מלעיל.
  
 
עולה מונוטונית:  
 
עולה מונוטונית:  
  
$ \frac{x_n}{x_{n-1}} = \frac{(1+\frac{1}{n})^{n}}{(1+\frac{1}{n-1})^{n-1}}=\frac{(\frac{n+1}{n})^{n}}{(\frac{n}{n-1})^{n-1}}=\frac{(n+1)^n (n-1)^{n-1}}{n^n n^{n-1}}=\frac{(n^2-1)^n n}{n^{2n}(n-1)}=(1-\frac{1}{n^2})^n \frac{n}{n-1} $
+
$$ \frac{x_n}{x_{n-1}} = \frac{(1+\frac{1}{n})^{n}}{(1+\frac{1}{n-1})^{n-1}}=\frac{(\frac{n+1}{n})^{n}}{(\frac{n}{n-1})^{n-1}}=\frac{(n+1)^n (n-1)^{n-1}}{n^n n^{n-1}}=$$
 +
$$\frac{(n^2-1)^n n}{n^{2n}(n-1)}=(1-\frac{1}{n^2})^n \frac{n}{n-1} $$
  
  
 
לפי אי שיוויון ברנולי, זה גדול או שווה לביטוי הבא:
 
לפי אי שיוויון ברנולי, זה גדול או שווה לביטוי הבא:
 
+
$$ (1+n(-\frac{1}{n^2})) \cdot \frac{n}{n-1} = \frac{(n-1)n}{n(n-1)} = 1 $$
$ (1+n(-\frac{1}{n^2})) \cdot \frac{n}{n-1} = \frac{(n-1)n}{n(n-1)} = 1 $
+
 
+
 
מכאן שזוהי סדרה מונוטונית עולה.
 
מכאן שזוהי סדרה מונוטונית עולה.
  
חסומה מלעיל:
+
\underline{חסומה מלעיל:}
  
 
$x_n=(1+\frac{1}{n})^n$
 
$x_n=(1+\frac{1}{n})^n$
  
 
ואם נפתח את הביטוי לפי הבינום של ניוטון נקבל
 
ואם נפתח את הביטוי לפי הבינום של ניוטון נקבל
 
+
$$x_n=1+\binom{n}{1} \frac{1}{n} + \binom {n}{2} \frac{1}{n^2} + \cdots + \frac{1}{n^n}$$
$x_n=1+\binom{n}{1} \frac{1}{n} + \binom {n}{2} \frac{1}{n^2} + \cdots + \frac{1}{n^n}$
+
 
+
 
איבר טיפוסי בסכום הזה הוא מהצורה
 
איבר טיפוסי בסכום הזה הוא מהצורה
 
+
$$\binom{n}{k} \frac{1}{n^k} = \frac{n!}{(n-k)! k!} \frac{1}{n^k} \leq \frac{n^n}{n^{n-k} k!} \frac{1}{n^k} = \frac{1}{k!} $$
$\binom{n}{k} \frac{1}{n^k} = \frac{n!}{(n-k)! k!} \frac{1}{n^k} \leq \frac{n^n}{n^{n-k} k!} \frac{1}{n^k} = \frac{1}{k!} $
+
 
+
 
ולכן
 
ולכן
 +
$$x_n \leq 2+\frac{1}{2!}+\frac{1}{3!}+\cdots +\frac{1}{n!}\leq 2+\frac{1}{2}+\frac{1}{4}+\cdots +\frac{1}{2^{n-1}} $$
  
$x_n \leq 2+\frac{1}{2!}+\frac{1}{3!}+\cdots +\frac{1}{n!}\leq 2+\frac{1}{2}+\frac{1}{4}+\cdots +\frac{1}{2^{n-1}} $
+
כל מה שאחרי ה-2 זה סדרה הנדסית אינסופית שסכומה 1 ולכן נקבל ש- $x_n\leq 3$
 
+
כל מה שאחרי ה-2 זה סדרה הנדסית אינסופית שסכומה 1 ולכן נקבל ש- $x_n<3$
+
  
 
אז זוהי סדרה מונוטונית עולה וחסומה מלעיל ולכן מתכנסת.  
 
אז זוהי סדרה מונוטונית עולה וחסומה מלעיל ולכן מתכנסת.  
 +
 +
\end{proof}
  
 
\subsection{תכונות של $ e $}
 
\subsection{תכונות של $ e $}
\underline{משפט:} $ \lim_{n\to \infty} \frac{1}{0!}+\frac{1}{1!}+\frac{1}{2!}+\cdots + \frac{1}{n!}=e $
+
\begin{thm}
\underline{הוכחה:} (ההוכחה משתמשת בכלים מסוף הקורס) ידוע ש- $e^x=\sum_{k=0}^\infty \frac{x^k}{k!} $ ולכן אם נציב $x=0$ נקבל את הדרוש.
+
$ \lim_{n\to \infty} \frac{1}{0!}+\frac{1}{1!}+\frac{1}{2!}+\cdots + \frac{1}{n!}=e $
 +
\end{thm}
 +
\begin{proof}
 +
נגדיר $e_n=\sum_{k=0}^n \frac{1}{k!} $ . צריך להראות ש- $e_n\to e $ . אם נשתמש באותה סדרה $x_n$ שהגדרנו אז ראינו בהוכחה של המשפט הקודם ש- $x_n\leq e_n $. מצד שני אם נכתוב את $x_n$ בצורה מפורשת אפשר גם לראות ש-
 +
$$x_n=1+1+\cdots + \frac{1\left ( 1-\frac{1}{n} \right ) \cdots \left ( 1-\frac{n-1}{n} \right )}{n!}$$
 +
ולכן אם נקבע $k$ ספציפי וניקח $n>k $ נקבל ש-
 +
$$x_n \geq 1+1+\cdots + \frac{1\left ( 1-\frac{1}{n} \right ) \cdots \left ( 1-\frac{k-1}{n} \right )}{k!}$$
 +
ואם נשאיף את $n\to \infty $ אז נקבל בצד ימין בדיוק את $e_k $ ובצד שמאל את $e$. גבול שומר על אי שיוויון חלש ולכן נקבל
 +
$$x_n\leq e_n \leq e$$
 +
ולפי משפט הסנדוויץ' נקבל את הדרוש.
  
$\\$
+
\end{proof}
  
 
נשים לב שאם נגדיר $e_n=\sum_{k=0}^n \frac{1}{k!} $ אז אם $N>n$ מתקיים  
 
נשים לב שאם נגדיר $e_n=\sum_{k=0}^n \frac{1}{k!} $ אז אם $N>n$ מתקיים  
  
$e_N-e_n=\frac{1}{(n+1)!}+\frac{1}{(n+2)!}+\cdots \frac{1}{N!}\leq \frac{1}{(n+1)!}+(\frac{1}{n+2}+\frac{1}{(n+2)^2}+\cdots + \frac{1}{(n+2)^{N-n-1}})=\frac{1}{(n+1)!}\cdot \frac{\frac{1}{n+2}((\frac{1}{n+2})^{N-n} - 1)}{\frac{1}{n+2} - 1 } \leq \frac{1}{(n+1)!}\frac{1}{1-\frac{1}{n+2}} = \frac{1}{(n+1)!}\frac{1}{n+1} $  
+
$$e_N-e_n=\frac{1}{(n+1)!}+\frac{1}{(n+2)!}+\cdots \frac{1}{N!}\leq \frac{1}{(n+1)!}+(\frac{1}{n+2}+\frac{1}{(n+2)^2}+\cdots + \frac{1}{(n+2)^{N-n-1}})=$$
 +
$$\frac{1}{(n+1)!}\cdot \frac{\frac{1}{n+2}((\frac{1}{n+2})^{N-n} - 1)}{\frac{1}{n+2} - 1 } \leq \frac{1}{(n+1)!}\frac{1}{1-\frac{1}{n+2}} = \frac{1}{(n+1)!}\frac{1}{n+1} $ $
  
 
כעת נשתמש בזה בשביל להוכיח:
 
כעת נשתמש בזה בשביל להוכיח:
  
\underline{משפט:} $e\not\in \mathbb{Q} $
+
\begin{thm}
 +
$e\not\in \mathbb{Q} $
 +
\end{thm}
  
\underline{הוכחה:}  
+
\begin{proof}
  
 
נניח $ e=\frac{p}{q} =1+\cdots +\frac{1}{n!}+\alpha_n $ ומכאן $|\alpha_n|<\frac{1}{(n+1)!}\frac{1}{n+1} $ וע"י כפל של 2 האגפים נקבל
 
נניח $ e=\frac{p}{q} =1+\cdots +\frac{1}{n!}+\alpha_n $ ומכאן $|\alpha_n|<\frac{1}{(n+1)!}\frac{1}{n+1} $ וע"י כפל של 2 האגפים נקבל
 
+
$$(n+1)!\frac{p}{q}=(n+1)!(1+\cdots+\frac{1}{n!})+(n+1)!\alpha_n $$
$(n+1)!\frac{p}{q}=(n+1)!(1+\cdots+\frac{1}{n!})+(n+1)!\alpha_n $
+
 
+
 
וזה נכון לכל $n$, בפרט ל- $n>q$ . במקרה זה, אגף שמאל שלם ואגף ימין מורכב ממשהו שהוא שלם ועוד $(n+1)!\alpha_n$ אבל החלק האחרון הזה הוא לא שלם משום שקטן מ- $\frac{1}{n+1} $. אגף שמאל, שהוא שלם הוא סכום של משהו שלם ומשהו שהוא לא שלם. סתירה
 
וזה נכון לכל $n$, בפרט ל- $n>q$ . במקרה זה, אגף שמאל שלם ואגף ימין מורכב ממשהו שהוא שלם ועוד $(n+1)!\alpha_n$ אבל החלק האחרון הזה הוא לא שלם משום שקטן מ- $\frac{1}{n+1} $. אגף שמאל, שהוא שלם הוא סכום של משהו שלם ומשהו שהוא לא שלם. סתירה
 
+
\end{proof}
<tex>קוד:זנב</tex>
+
</latex2pdf>
+

גרסה אחרונה מ־20:16, 4 באוקטובר 2014

\subsection{הגדרה} נגדיר את $x_n=(1+\frac{1}{n})^n$ .

\begin{thm} קיים הגבול $\lim_{n\to \infty} x_n $ , לגבול הזה קוראים $e$. \end{thm}

\begin{proof} נוכיח שהסדרה מונוטונית עולה וחסומה מלעיל.

עולה מונוטונית:

$$ \frac{x_n}{x_{n-1}} = \frac{(1+\frac{1}{n})^{n}}{(1+\frac{1}{n-1})^{n-1}}=\frac{(\frac{n+1}{n})^{n}}{(\frac{n}{n-1})^{n-1}}=\frac{(n+1)^n (n-1)^{n-1}}{n^n n^{n-1}}=$$ $$\frac{(n^2-1)^n n}{n^{2n}(n-1)}=(1-\frac{1}{n^2})^n \frac{n}{n-1} $$


לפי אי שיוויון ברנולי, זה גדול או שווה לביטוי הבא: $$ (1+n(-\frac{1}{n^2})) \cdot \frac{n}{n-1} = \frac{(n-1)n}{n(n-1)} = 1 $$ מכאן שזוהי סדרה מונוטונית עולה.

\underline{חסומה מלעיל:}

$x_n=(1+\frac{1}{n})^n$

ואם נפתח את הביטוי לפי הבינום של ניוטון נקבל $$x_n=1+\binom{n}{1} \frac{1}{n} + \binom {n}{2} \frac{1}{n^2} + \cdots + \frac{1}{n^n}$$ איבר טיפוסי בסכום הזה הוא מהצורה $$\binom{n}{k} \frac{1}{n^k} = \frac{n!}{(n-k)! k!} \frac{1}{n^k} \leq \frac{n^n}{n^{n-k} k!} \frac{1}{n^k} = \frac{1}{k!} $$ ולכן $$x_n \leq 2+\frac{1}{2!}+\frac{1}{3!}+\cdots +\frac{1}{n!}\leq 2+\frac{1}{2}+\frac{1}{4}+\cdots +\frac{1}{2^{n-1}} $$

כל מה שאחרי ה-2 זה סדרה הנדסית אינסופית שסכומה 1 ולכן נקבל ש- $x_n\leq 3$

אז זוהי סדרה מונוטונית עולה וחסומה מלעיל ולכן מתכנסת.

\end{proof}

\subsection{תכונות של $ e $} \begin{thm} $ \lim_{n\to \infty} \frac{1}{0!}+\frac{1}{1!}+\frac{1}{2!}+\cdots + \frac{1}{n!}=e $ \end{thm} \begin{proof} נגדיר $e_n=\sum_{k=0}^n \frac{1}{k!} $ . צריך להראות ש- $e_n\to e $ . אם נשתמש באותה סדרה $x_n$ שהגדרנו אז ראינו בהוכחה של המשפט הקודם ש- $x_n\leq e_n $. מצד שני אם נכתוב את $x_n$ בצורה מפורשת אפשר גם לראות ש- $$x_n=1+1+\cdots + \frac{1\left ( 1-\frac{1}{n} \right ) \cdots \left ( 1-\frac{n-1}{n} \right )}{n!}$$ ולכן אם נקבע $k$ ספציפי וניקח $n>k $ נקבל ש- $$x_n \geq 1+1+\cdots + \frac{1\left ( 1-\frac{1}{n} \right ) \cdots \left ( 1-\frac{k-1}{n} \right )}{k!}$$ ואם נשאיף את $n\to \infty $ אז נקבל בצד ימין בדיוק את $e_k $ ובצד שמאל את $e$. גבול שומר על אי שיוויון חלש ולכן נקבל $$x_n\leq e_n \leq e$$ ולפי משפט הסנדוויץ' נקבל את הדרוש.

\end{proof}

נשים לב שאם נגדיר $e_n=\sum_{k=0}^n \frac{1}{k!} $ אז אם $N>n$ מתקיים

$$e_N-e_n=\frac{1}{(n+1)!}+\frac{1}{(n+2)!}+\cdots \frac{1}{N!}\leq \frac{1}{(n+1)!}+(\frac{1}{n+2}+\frac{1}{(n+2)^2}+\cdots + \frac{1}{(n+2)^{N-n-1}})=$$ $$\frac{1}{(n+1)!}\cdot \frac{\frac{1}{n+2}((\frac{1}{n+2})^{N-n} - 1)}{\frac{1}{n+2} - 1 } \leq \frac{1}{(n+1)!}\frac{1}{1-\frac{1}{n+2}} = \frac{1}{(n+1)!}\frac{1}{n+1} $ $

כעת נשתמש בזה בשביל להוכיח:

\begin{thm} $e\not\in \mathbb{Q} $ \end{thm}

\begin{proof}

נניח $ e=\frac{p}{q} =1+\cdots +\frac{1}{n!}+\alpha_n $ ומכאן $|\alpha_n|<\frac{1}{(n+1)!}\frac{1}{n+1} $ וע"י כפל של 2 האגפים נקבל $$(n+1)!\frac{p}{q}=(n+1)!(1+\cdots+\frac{1}{n!})+(n+1)!\alpha_n $$ וזה נכון לכל $n$, בפרט ל- $n>q$ . במקרה זה, אגף שמאל שלם ואגף ימין מורכב ממשהו שהוא שלם ועוד $(n+1)!\alpha_n$ אבל החלק האחרון הזה הוא לא שלם משום שקטן מ- $\frac{1}{n+1} $. אגף שמאל, שהוא שלם הוא סכום של משהו שלם ומשהו שהוא לא שלם. סתירה \end{proof}